www.vorkurse.de
Ein Projekt von vorhilfe.de
Die Online-Kurse der Vorhilfe

E-Learning leicht gemacht.
Hallo Gast!einloggen | registrieren ]
Startseite · Mitglieder · Teams · Forum · Wissen · Kurse · Impressum
Forenbaum
^ Forenbaum
Status Mathe-Vorkurse
  Status Organisatorisches
  Status Schule
    Status Wiederholung Algebra
    Status Einführung Analysis
    Status Einführung Analytisc
    Status VK 21: Mathematik 6.
    Status VK 37: Kurvendiskussionen
    Status VK Abivorbereitungen
  Status Universität
    Status Lerngruppe LinAlg
    Status VK 13 Analysis I FH
    Status Algebra 2006
    Status VK 22: Algebra 2007
    Status GruMiHH 06
    Status VK 58: Algebra 1
    Status VK 59: Lineare Algebra
    Status VK 60: Analysis
    Status Wahrscheinlichkeitst

Gezeigt werden alle Foren bis zur Tiefe 2

Navigation
 Startseite...
 Neuerdings beta neu
 Forum...
 vorwissen...
 vorkurse...
 Werkzeuge...
 Nachhilfevermittlung beta...
 Online-Spiele beta
 Suchen
 Verein...
 Impressum
Das Projekt
Server und Internetanbindung werden durch Spenden finanziert.
Organisiert wird das Projekt von unserem Koordinatorenteam.
Hunderte Mitglieder helfen ehrenamtlich in unseren moderierten Foren.
Anbieter der Seite ist der gemeinnützige Verein "Vorhilfe.de e.V.".
Partnerseiten
Weitere Fächer:

Open Source FunktionenplotterFunkyPlot: Kostenloser und quelloffener Funktionenplotter für Linux und andere Betriebssysteme
Forum "Stochastik" - Varianz
Varianz < Stochastik < Oberstufe < Schule < Mathe < Vorhilfe
Ansicht: [ geschachtelt ] | ^ Forum "Stochastik"  | ^^ Alle Foren  | ^ Forenbaum  | Materialien

Varianz: lin. Transformation
Status: (Frage) beantwortet Status 
Datum: 19:37 Do 30.06.2011
Autor: mikexx

Aufgabe
Mal eine Frage zu einer Eigenschaft der Varianz; es gilt ja:

Var(aX+b)=a^2Var(X)

Soweit, so gut.

Nun habe ich eine Aufgabe, bei der [mm] X_i [/mm] unabhängige, identisch verteilte Zufallsvariablen sind und [mm] M=\frac{1}{n}\sum_{i=1}^{n}X_i [/mm] der Mittelwert dieser Zufallsvariablen.

Gilt:

[mm] Var(X_i-M)=Var(X_i), [/mm] i=1,...,n

??

..

        
Bezug
Varianz: Antwort
Status: (Antwort) fertig Status 
Datum: 21:08 Do 30.06.2011
Autor: luis52

Moin


> Gilt:
>  
> [mm]Var(X_i-M)=Var(X_i),[/mm] i=1,...,n
>  

Nein. Betrachte

[mm] $\text{Var}\left[X_i-\frac{1}{n}\sum_{j=1}^{n}X_j\right]=\text{Var}\left[(1-\frac{1}{n})X_i-\frac{1}{n}\sum_{j\ne i}^{n}X_j\right] [/mm] $ ...

vg Luis    

Bezug
                
Bezug
Varianz: Frage (beantwortet)
Status: (Frage) beantwortet Status 
Datum: 21:13 Do 30.06.2011
Autor: mikexx

Ich soll nämlich

[mm] E(\sum_{i=1}^{n}(X_i-M)^2) [/mm] berechnen und weiß dann nicht so wirklich, wie ichs machen kann.

Erstmal vielleicht die einzelnen Summanden aufschreiben??

= [mm] E[(X_1-M)^2]+...+E[(X_n-M)^2] [/mm] ??

Und das ist doch jetzt das Gleiche wie

= [mm] Var(X_1-M)+E(X_1-M)^2+...+Var(X_n-M)+E(X_n-M)^2 [/mm]

Okay? Und wie gehts weiter? [Falls es bis hier stimmt...]

Bezug
                        
Bezug
Varianz: Antwort
Status: (Antwort) fertig Status 
Datum: 22:04 Do 30.06.2011
Autor: luis52

In meinem Lieblings-Statistikbuch

@BOOK{Mood74,
  title = {Introduction to the Theory of Statistics},
  publisher = {Mc-Graw-Hill},
  year = {1974},
  author = {A. M. Mood and F. A. Graybill and D. C. Boes},
  edition = {3.}
}

finde ich auf Seite 239-240 die folgenden Tipps. Sei [mm] $\mu=\text{E}[X_i]$ [/mm] und [mm] $\sigma^2=\text{Var}[X_i]$: [/mm]

1) [mm] $\sum_{i=1}^{n}(X_i-\mu)^2=\sum_{i=1}^{n}(X_i-M)^2+n (M-\mu)^2$ [/mm]

2) [mm] $\text{E}[\frac{1}{n-1}\sum_{i=1}^{n}(X_i-M)^2]=\sigma^2$. [/mm]

vg Luis      

PS: Es waere schoen, wenn du kuenftig die eigentliche Fragestellung nicht in homoeopathischen Dosen mitteilst:

> Ich soll nämlich  [mm]E(\sum_{i=1}^{n}(X_i-M)^2)[/mm] berechnen

Bezug
                                
Bezug
Varianz: Frage (beantwortet)
Status: (Frage) beantwortet Status 
Datum: 13:07 Fr 01.07.2011
Autor: mikexx

Hallo, ich nochmal.

Da hat jemand die gleiche Aufgabe wie ich zu bearbeiten (nur noch zusätzlich ist dort gegeben, dass [mm] E(X_i)=0 [/mm] und [mm] Var(X_i)=v) [/mm]

Ich weiß, ihr seht es nicht so gerne, wenn man hier Links zu anderen Foren gibt, aber die Rechnung dort ist so ellenlang, dass ich das nicht alles hier aufschreiben möchte.

Ich meine da den vorletzten Beitrag.

http://www.matheboard.de/thread.php?postid=1431626#post1431626


Stimmt die Rechnung dort?

Bezug
                                        
Bezug
Varianz: Antwort
Status: (Antwort) fertig Status 
Datum: 13:40 Fr 01.07.2011
Autor: luis52


>
> Stimmt die Rechnung dort?

Nein.

vg Luis


Bezug
                                                
Bezug
Varianz: Frage (beantwortet)
Status: (Frage) beantwortet Status 
Datum: 13:53 Fr 01.07.2011
Autor: mikexx

Okay, die ersten Zeilen sehen für mich aber ganz plausibel aus, wo ist dort der Fehler?

Bzw. ab wo wird es dort falsch?

Bis zu diesem Punkt erscheint es mir korrekt:



[mm]=n\cdot \left[v-\frac{2}{n}\cdot E\left(X_i\cdot \sum_{i=1}^{n}X_i\right)+\frac{1}{n^2}\cdot E\left(\left(\sum_{i=1}^{n}X_i\right)^2\right)\right][/mm]

Bezug
                                                        
Bezug
Varianz: Frage (beantwortet)
Status: (Frage) beantwortet Status 
Datum: 14:58 Fr 01.07.2011
Autor: mikexx

Nach Deinem Tipp oben muss ja herauskommen:

[mm]v\cdot (n-1)[/mm].


Ich habe dem Fragesteller, dessen Link ich eben gegeben habe, das gesagt und er hat das jetzt auch heraus!

Ist die Rechnung jetzt korrekt?

Siehe letzter Beitrag:

http://www.matheboard.de/thread.php?postid=1431699#post1431699

Bezug
                                                                
Bezug
Varianz: hat sich erledigt..
Status: (Mitteilung) Reaktion unnötig Status 
Datum: 15:23 Fr 01.07.2011
Autor: mikexx

In dem Forum hat schon jemand gesagt, dass die Rechnung jetzt stimmt.

:-)

Damit hat sich meine letzte Frage eigentlich erledigt.

Bezug
Ansicht: [ geschachtelt ] | ^ Forum "Stochastik"  | ^^ Alle Foren  | ^ Forenbaum  | Materialien


^ Seitenanfang ^
www.vorkurse.de
[ Startseite | Mitglieder | Teams | Forum | Wissen | Kurse | Impressum ]